Brienne stated that 0.33 and
1/3 are equivalent. Do you agree? Explain why or why
not.

Answers

Answer 1

Answer:

it is not

Step-by-step explanation:

0.33 is rounded off whereas 1/3 would have a continued decimal of 3, i.e. 0.333333333


Related Questions

−4x + 3y =7
5x − 2y =35
What is the solution?

Answers

Answer:   x = 39/10     y = 29/5

Answer:

(17,25)

Step-by-step explanation:

To find the solution to the system, solve with elimination.

Equation 1.

−4x + 3y =7

Equation 2.

5x − 2y =35

To cancel out the y-variable, multiply the first equation by 2, and the second equation by 3.

2(−4x + 3y =7)

-8x+6y=14

3(5x − 2y =35)

15x-6y=105

Combine both equations.

-8x+6y=14

15x-6y=105

7x=119

x= 17

Plug in x into the first equation to solve for y.

-8x+6y=14

-8(17)+6y=14

-136+6y=14

6y=150

y=25

9th grade math please help will give brainliest !!!

Answers

Answer:

48 g - 40 h + 64

Step-by-step explanation:

you take the 8 and multiply it by the numbers that are in the paraentheses. keep the variables the same

Juan has 20 books to sell. He sells the books for $15 each.

The amount of money Juan makes from selling x books is represented by the equation y=15x
y
=
15
x



What is the theoretical range of the equation?

Question 2 options:

All multiples of 20 between 0 and 300, inclusive


All integers from 0 to 20, inclusive


All multiples of 15 between 0 and 300, inclusive


All real numbers

Answers

Answer:

Well u would multiply 20 x 15 to find out the highest if the range which is 300 . So that knocks it down to B or D. But since they're $15 each, It would be based of the multiples of 15 making your answer B .

The range is all the y values.....or the f(x) values .

So the practical range would be the multiples of 15 between 0 and 300,Inclusive .

Hope this helps you!!

PLEASEEE HELPPPP ASAPPPP

Answers

[tex]\cos 27^{\circ} = \dfrac{KL}x\\\\\implies \cos 27^{\circ} = \dfrac{6.3}x\\\\\implies x = \dfrac{6.3}{\cos 27^{\circ}}=7.1~ \text{units.}[/tex]

Can the distributive property be used to rewrite
4x (6 - 3)?

Answers

Answer:

12x

Step-by-step explanation:

cuz i said so

I really need help, this is probability and statistics, if anyone else is taking this class, please help, it is a unit test study guide.

Answers

Answer:

id like to help but the picture is really blurry so im afraid that i cant. but i know there are others who can!

Step-by-step explanation:

i hope this helps you!

I can't see the picture good

HELP!!!

n – 3 = ‐1

Please do not just type in the answer please explain how you got your answer! Whoever explains (CORRECTLY) first gets brainliest!

Answers

ANSWER:n=2

EXPLANATION: n-3=-1
n= -1+3
n=2

Answer:

n = 2

Step 1: Add 3 to both sides to isolate x.

n – 3 = ‐1

Step 2: Solve

=> n = -1 + 3

Final Answer: Answer to the question.

=> n = 2

Therefore, n = 2 is our answer.

Hoped this helped.

Find the domain of the expression.

the square root of x+5

Answers

See attachment for answer and math work.

Answer:

Step-by-step explanation:

"Domain" means what numbers are allowed to be x (or on a graph, for what x's does the graph exist)

Since inside a square root you have to have a positive number (actually, just not a negative number). So all the positive numbers work (any positive+5 is a positive number) BUT ALSO x can be 0, BUT ALSO, x can be as small as -5 and still the value inside the square root will not be negative

SO THIS MEANS all the x's we're allowed to use (that's the DOMAIN) are everything from -5 and bigger. In math we write that as x>= -5

There are two other main ways to write it: [-5, infinity symbol) OR {x|x>=-5} these all mean the same thing.

Last week the number of bacteria in a petri dish was 2.25×
×
102
10
2
. This week the number of bacteria in the petri dish is 2.75×
×
105
10
5
. How many times more bacteria are in the petri dish this week?
Round the decimal to the nearest tenth.

Answers

Answer:

1221.2 factor increase

Step-by-step explanation:

It is difficult interpreting the numbers.  I'll assume:

2.25x10^2 and 2.75x10^5

----

225

275000

The difference(increase) is 274775, or 2.75 x 10^5 to 3 sig figs

There is a (2.75 x 10^5)/(2.25x10^2) or 1221.2 factor increase in bacteria between the two weeks.

Graph
Y-7=-3/5-(x+2)

Answers

Answer:

See attached

Step-by-step explanation:

A simpler form of the same equation is:

y = -4 +(4+2/5)

Solve the system of inequalities by graphing.
12x+6y≥15
2x-4y>8

Answers

Answer:

?

Step-by-step explanation:

Hello, Can you please take a look at this screenshot I have below, This problem is confusing :/

Answers

Answer:

See answers below.

Step-by-step explanation:

m ∠A + m ∠B + m ∠C = 180°          The sum of the angles in a triangle = 180

28°  +   x - 4   + 2x  = 180°              Substitution

x =  52°                                            Algebra

28°  +  x - 4  + 2x  = 180°

    24° + 3x = 180°

     3x = 156°

      x = 156/3

       x = 52°

Check:   28° + 48 + 104°  = 180√

The railroad company wants to sell $8,134 in tickets. If each ticket costs $7, how many
tickets will the railroad have to sell to meet its goal?

Answers

Answer:

1162 tickets will need to be sold

Step-by-step explanation:

8134 / 7 = 1,162

Sally is making 15 individual blueberry cheesecakes. How many pounds of blueberries will be in each cheesecake if 514 pounds of blueberries are divided equally among them?

Answers

Answer:

About 35 or 34.27 if you round to the tenths spot.

Step-by-step explanation:

514 divided by 15 is 34.27 once rounded.

What is 1+1 because it's obvious that it's 3!!! Duh!!! So hehe

Answers

Answer:

IT'S TWOOOOOOOOOOOOOOOO 222222222222222

Line c passes through points (9, 10) and (1, 3). Line d is perpendicular to c. What is the slope
of line d?

Answers

Answer:

-8/7

Step-by-step explanation:

First, find the slope of line c.

(x1, y1) = (9, 10)

(x2, y2) = (1, 3)

The slope formula is:

[tex]m=\frac{y2-y1}{x2-x1}[/tex]

Solve for the slope using the given values:

[tex]m=\frac{3-10}{1-9}\\\\m=\frac{-7}{-8}\\\\m=\frac{7}{8}[/tex]

The slope of line c is 7/8. To find the slope perpendicular to it, you need to find the negative reciprocal of that number. An example of that would be 2 and -1/2. Basically, flip the fraction and the +/-.

In this case, the negative reciprocal of 7/8 is -8/7. The image below doesn't account for the y-intercept, but it does show that the slopes are perpendicular.

Answer:

slope = - [tex]\frac{8}{7}[/tex]

Step-by-step explanation:

Calculate the slope m of line c using the slope formula

m = [tex]\frac{y_{2}-y_{1} }{x_{2}-x_{1} }[/tex]

with (x₁, y₁ ) = (9, 10 ) and (x₂, y₂ ) = (1, 3 )

m = [tex]\frac{3-10}{1-9}[/tex] = [tex]\frac{-7}{-8}[/tex] = [tex]\frac{7}{8}[/tex]

Given a line with slope m then the slope of a line perpendicular to it is

[tex]m_{perpendicular}[/tex] = - [tex]\frac{1}{m}[/tex] = - [tex]\frac{1}{\frac{7}{8} }[/tex] = - [tex]\frac{8}{7}[/tex] ← slope of line d

write an expression describing all the angles that are coterminal with 266°

Answers

Answer:

-814°, -454°, -94°, 266°, 626°. Did you mean like that?

Jackson’s rectangular bedroom has an area of 90 square feet. The area of his bedroom is 9 times that of his rectangular closet. If the closet is 2 feet wide. What is it’s length?

Answers

Answer:

Im pretty sure the answer is 5.

Step-by-step explanation:

in the expression 3² + 5 · 3 what needs to be done first? exponents addition multiplication

Answers

Answer:

If you use the PEMDAS method, you can see that exponents should be done first.

Step-by-step explanation:

Mariah made a down payment on a van. Which of the following is true because of the down payment?
A. She has additional time to repay the loan.
B. Her credit limit will increase
C. She has some equity or ownership right away.
D. Her insurance will decrease.

Answers

The answer should be C. :)

Answer:

Option C. She has some equity or ownership right away.

Step-by-step explanation:

I did the Exam too. And got it right. Hope this helped.

Factorise 9a³ - 27a² - 100a + 300, if it is given that (3a + 10) is a factor of it.​

Answers

Answer:

(3a + 10)(3a - 10(a - 3).

Step-by-step explanation:

Do a long division:

3a + 10 ) 9a^3 - 27a^2 - 100a + 300 ( 3a^2 - 19a + 30 <----- Quotient

              9a^3 + 30a^2

                        -57a^2 - 100a

                        -57a^2 - 190a

                                       90a + 300

                                       90a + 300

                                        ..................

Factorise the Quotient:

3a^2 - 19a + 30

= 3a^2 - 9a - 10a + 30

= 3a(a - 3) - 10(a - 3)

= (3a - 10)(a - 3)

HELP PLEASEEEE what is the range of 0,5,10,15,20,25
5

11

20

25

Answers

Answer:

25!

Step-by-step explanation:

Subtract the highest and lowest so its 25:)

Answer:

25

Step-by-step explanation:

Range is the largest number minus the lowest

PLEASE ANSWER FAST....Which expressions have a value equal to or greater than 13?

I WILL MARK BRAINLIEST

Answers

Answer: D and B

Step-by-step explanation:

D: 10/10 can be simplified to 1/1, or 1. 1 times 13 is equal to 13.

B: 4 x 13 is simply 52.

Hope this helps!

A. 5/6 x 13 = 10.83

B. 4 x 13 = 52

C. 3/4 x 13 = 9.75

D. 10/10 (1) x 13 = 13

B. 4 x 13 & D. 10/10 (1) x 13 are your answers.

Which statement is true about the function f(x)=(1/3)^x and g(x)=3^-x?
a. The function have the same graph
b. The graph of the function is not the same
c. The graph of the function is not exponential

Answers

the answer should be B


In a scale drawing, 3 millimeters represents 17 meters. Exactly how many millimeters represents 55 meters?

Answers

Answer:

See image

Step-by-step explanation:

Compare "like" values to write a proportion (two fractions or ratios set equal to each other) Then you can cross multiply to solve the equation for x

Please help worth 20 points

Answers

Answer:

Try going on D.e.s.m.o.s.c.o.m it worked for me when I  had something like this

Step-by-step explanation:

GURL POWER

Which table is y a function of x?

Answers

Answer:

The first chart

Step-by-step explanation:

g(x) = 3x + 1; Find g(7)

Answers

g(7)=3(7)+1
g(7)=21+1
g(7)=22

3xy divide 18 ,
x=
Y=

Answers

The answer is should be
1
——xy
6

Please hurry and help

15 points

Answers

Answer:

A

Step-by-step explanation:

Seems like a reasonable answer choice

Other Questions
What did he study every day as a child, the Torah in Hebrew or the History of the Roman Empire in Latin? What reptilian taxon is extant (has at least some members that are still living; opposite of extinct) What is photosynthesis? can someone tell me what this is 6% OF 56% X 60% X 40% how long does it take to recover from cataract surgery Find the slope. Use point-slope formula to write the equation of the line described. Simplify your equation to slope-intercept form.Through (5,2), Perpendicular to y=-5/7x The mass of 100 tins of corned beef is 34 g. What is the mass, in grams (g), of 1 tin of corned beef? Como se faz o clculo 135,2 4) Eric bought a canoe for $333.75. He bought a life vest for $134.87 and an ore paddle for $67.89The sales tax was 5.25%. What was his total cost? Find the equation of a line parallel to line y = 6 that passes through the point (-2,-1). Compared with central America, Caribbean culture is more influenced by which of the following ethnic groups if anyone knows Spanish again I need more help please! it's :1. 235 34 hasta milesimos 2. 235. 34 hasta diezmilesimos Five pipes can fill a tank in 80 minutes. At the same rate, how long will it take 8 pipes to fill the same tank? how do i multiply 4000x by the square root of 42 13 fasteners cost 78 cents find the cost for purchasing 4, 687 fasteners What power would you multiply 22 by to get an answer of 22,000? 10^3? 10^5 10^2 10^4 also i dont know how to use this app. someone help me with this its literally easy but im still having a hard time to do it A triangle has vertices at A(-3, 2), B(-1, -5) and C(4, 5). The triangle is transformed using the rule ro,o . What are the coordinates of A", B', and C? 38499 divided by 779 Explain the difference between The Red Army and The White Army?